How does Ehrenfest's theorem apply to the quantum harmonic oscillator?
$begingroup$
Ehrenfest's theorem, to my level of understanding, says that expectation values for quantum mechanical observables obey their Newtonian mechanics counterparts, which means that we can use newton's laws on expectation values. However, in the case of the quantum harmonic oscillator, this clearly does not look newtonian because the expectation value of the position does not oscillate like the newtonian $sinomega t$.
These states are of the form $psi=K(n, xi)e^{-xi^2/2}$. Why do they not obey Ehrenfest's theorem? They don't give a harmonic oscillator, imo.
quantum-mechanics classical-mechanics wavefunction harmonic-oscillator
New contributor
$endgroup$
add a comment |
$begingroup$
Ehrenfest's theorem, to my level of understanding, says that expectation values for quantum mechanical observables obey their Newtonian mechanics counterparts, which means that we can use newton's laws on expectation values. However, in the case of the quantum harmonic oscillator, this clearly does not look newtonian because the expectation value of the position does not oscillate like the newtonian $sinomega t$.
These states are of the form $psi=K(n, xi)e^{-xi^2/2}$. Why do they not obey Ehrenfest's theorem? They don't give a harmonic oscillator, imo.
quantum-mechanics classical-mechanics wavefunction harmonic-oscillator
New contributor
$endgroup$
$begingroup$
Maybe this question of mine can answer your question: physics.stackexchange.com/questions/267835/…
$endgroup$
– Quantumwhisp
15 hours ago
$begingroup$
Though it does not address the question asked, I used to have students plot the position probability distribution for the quantum and classical cases over a range of energies corresponding to, say, $n = 1,2,3,4,10$ and a "big" number like $20$. I'll post a figure in chat: chat.stackexchange.com/transcript/message/49407141#49407141.
$endgroup$
– dmckee♦
2 hours ago
add a comment |
$begingroup$
Ehrenfest's theorem, to my level of understanding, says that expectation values for quantum mechanical observables obey their Newtonian mechanics counterparts, which means that we can use newton's laws on expectation values. However, in the case of the quantum harmonic oscillator, this clearly does not look newtonian because the expectation value of the position does not oscillate like the newtonian $sinomega t$.
These states are of the form $psi=K(n, xi)e^{-xi^2/2}$. Why do they not obey Ehrenfest's theorem? They don't give a harmonic oscillator, imo.
quantum-mechanics classical-mechanics wavefunction harmonic-oscillator
New contributor
$endgroup$
Ehrenfest's theorem, to my level of understanding, says that expectation values for quantum mechanical observables obey their Newtonian mechanics counterparts, which means that we can use newton's laws on expectation values. However, in the case of the quantum harmonic oscillator, this clearly does not look newtonian because the expectation value of the position does not oscillate like the newtonian $sinomega t$.
These states are of the form $psi=K(n, xi)e^{-xi^2/2}$. Why do they not obey Ehrenfest's theorem? They don't give a harmonic oscillator, imo.
quantum-mechanics classical-mechanics wavefunction harmonic-oscillator
quantum-mechanics classical-mechanics wavefunction harmonic-oscillator
New contributor
New contributor
edited 10 hours ago
knzhou
44.8k11122216
44.8k11122216
New contributor
asked 16 hours ago
SodaSoda
544
544
New contributor
New contributor
$begingroup$
Maybe this question of mine can answer your question: physics.stackexchange.com/questions/267835/…
$endgroup$
– Quantumwhisp
15 hours ago
$begingroup$
Though it does not address the question asked, I used to have students plot the position probability distribution for the quantum and classical cases over a range of energies corresponding to, say, $n = 1,2,3,4,10$ and a "big" number like $20$. I'll post a figure in chat: chat.stackexchange.com/transcript/message/49407141#49407141.
$endgroup$
– dmckee♦
2 hours ago
add a comment |
$begingroup$
Maybe this question of mine can answer your question: physics.stackexchange.com/questions/267835/…
$endgroup$
– Quantumwhisp
15 hours ago
$begingroup$
Though it does not address the question asked, I used to have students plot the position probability distribution for the quantum and classical cases over a range of energies corresponding to, say, $n = 1,2,3,4,10$ and a "big" number like $20$. I'll post a figure in chat: chat.stackexchange.com/transcript/message/49407141#49407141.
$endgroup$
– dmckee♦
2 hours ago
$begingroup$
Maybe this question of mine can answer your question: physics.stackexchange.com/questions/267835/…
$endgroup$
– Quantumwhisp
15 hours ago
$begingroup$
Maybe this question of mine can answer your question: physics.stackexchange.com/questions/267835/…
$endgroup$
– Quantumwhisp
15 hours ago
$begingroup$
Though it does not address the question asked, I used to have students plot the position probability distribution for the quantum and classical cases over a range of energies corresponding to, say, $n = 1,2,3,4,10$ and a "big" number like $20$. I'll post a figure in chat: chat.stackexchange.com/transcript/message/49407141#49407141.
$endgroup$
– dmckee♦
2 hours ago
$begingroup$
Though it does not address the question asked, I used to have students plot the position probability distribution for the quantum and classical cases over a range of energies corresponding to, say, $n = 1,2,3,4,10$ and a "big" number like $20$. I'll post a figure in chat: chat.stackexchange.com/transcript/message/49407141#49407141.
$endgroup$
– dmckee♦
2 hours ago
add a comment |
3 Answers
3
active
oldest
votes
$begingroup$
It is actually true, in an almost trivial way. The Ehrenfest theorem states that,
begin{equation}
frac{d}{dt}langle xrangle=langle prangle,quad frac{d}{dt}langle prangle =- langle V'(x)rangle
end{equation}
However for all eigenfunctions for the Harmonic oscillator $langle xrangle=0$ (and therefore $langle V'(x)rangle=0$) and $langle prangle=0$. So the Ehrenfest theorem on the eigenstates reduces to $0=0$.
You can see that the general version of the Ehrenfest theorem works trivially for all eigenstates. It states that for the arbitrary observable $A$ its expectation value satisfies the equation,
begin{equation}
frac{d}{dt} langle Arangle=frac{1}{ihbar}langle [A,H]rangle+langle frac{partial A}{partial t}rangle
end{equation}
However on the eigenstates,
begin{equation}
langlepsi_n| [A,H]|psi_nrangle=langlepsi_n|AH-HA|psi_nrangle=E_nlanglepsi_n|A-A|psi_nrangle=0
end{equation}
So the expectation value of the observable that doesn't explicitly depend on time, doesn't evolve on the eigenstates which is what you would expect.
So where does the Ehrenfest theorem lead to the classical dynamics? You need to consider the localized wavepackets. The simplest example would be the coherent state of the Harmonic oscillator that is the Gaussian wavepacket that follows the classical trajectory
For the Harmonic oscillator the Ehrenfest theorem is always "classical" if only in a trivial way (as in case of the eigenstates). However in general the Ehrenfest theorem reduces to the classical equation of motion only on such localized wavepackets that concentrate near the classical trajectory as $hbar$ goes to zero. The key point happens to be the interchange $langle V'(x)rangle mapsto V'(langle xrangle)$ that on general states can't be done. So if you want to recover some classical dynamics from the quantum theory look on the localized wavepackets.
$endgroup$
add a comment |
$begingroup$
- Your version of $psi$ is (I'm sure you know) derived from the Time Independent Schrodinger equation, $hat{H}psi=Epsi$. To find time-dependent solutions, we solve $$ifrac{partial}{partial t}psi=hat{H}psi.$$ You were trying to solve for stationary states, and the entire point of those is that $|psi(x)|^2$ does not change over time. Still, for these stationary solutions solutions,$$frac{mathrm{d}}{mathrm{d}t}left<xright>=left<pright>=0; frac{mathrm{d}}{mathrm{d}t}left<pright>=-left<frac{mathrm{d}}{mathrm{d}x}V (x)right>=0,$$ which is in accordance with the Ehrenfest theorem (albeit uninformatively).
Be careful about the time dependence of your reported $psi$: you're actually dealing with $Psi(x, t)=psi(x)e^{-itE/hbar}$ for those stationary states. Of course, this doesn't relate to the Ehrenfest theorem, but it's something worth mentioning: the complex and real parts are oscillating, as shown by the pink and blue lines in this diagram (from the wikipedia page on QHO):
Do not make the mistake of suggesting that all derivatives with respect to time are necessarily automatically equal to zero because the wavefunction is time-independent. Contrary to what the shorthand notation suggests, we do have a (separable) time-dependent bit.
Referring to the same diagram, observe parts G and H: these represent coherent states, which can be understood using Ehrenfest's theorem because $|psi|^2$ looks like a Gaussian which follows a classical $sin$ or $cos$ function.
See
Kanasugi, H., and H. Okada. “Systematic Treatment of General Time-Dependent Harmonic Oscillator in Classical and Quantum Mechanics.” Progress of Theoretical Physics, vol. 93, no. 5, 1995, pp. 949–960., doi:10.1143/ptp/93.5.949.
$endgroup$
add a comment |
$begingroup$
What you wrote down is just a complete set of solutions of the time independent Schrödinger equation.
begin{align}
[- frac{hbar²}{2m} Delta + V(x) ] Psi(x) = E Psi(x)
end{align}
Of course this solutions don't carry any time dependence, because the time-independent Schrödinger equation (in this representation) only makes statements about functions that depend on the spatial coordinates only (in this case, this is only x).
How to get to the time-dependent solutions?
A particular property of the solutions $Psi_{E}$ of the time-independent Schrödinger Equation is that $Psi_{E}(x) e^{-i frac{E}{hbar}t}$ is a solution to the time-dependent Schrödinger equation.
If you would apply this to your suggested set of solutions of the harmonic oscillator, you would arrive at time dependent solutions. THOSE are the ones that the Ehrenfest-Theorem makes a statement about.
You would then calculate that the expectation values $<X>$ and $<P>$ do not change. But you would as well calculate that both of this values are 0. This is in perfect agreement with the Theorems of Ehrenfest, and the classical analogon would be a particle resting at the deepest spot of the harmonic potential.
$endgroup$
add a comment |
Your Answer
StackExchange.ifUsing("editor", function () {
return StackExchange.using("mathjaxEditing", function () {
StackExchange.MarkdownEditor.creationCallbacks.add(function (editor, postfix) {
StackExchange.mathjaxEditing.prepareWmdForMathJax(editor, postfix, [["$", "$"], ["\\(","\\)"]]);
});
});
}, "mathjax-editing");
StackExchange.ready(function() {
var channelOptions = {
tags: "".split(" "),
id: "151"
};
initTagRenderer("".split(" "), "".split(" "), channelOptions);
StackExchange.using("externalEditor", function() {
// Have to fire editor after snippets, if snippets enabled
if (StackExchange.settings.snippets.snippetsEnabled) {
StackExchange.using("snippets", function() {
createEditor();
});
}
else {
createEditor();
}
});
function createEditor() {
StackExchange.prepareEditor({
heartbeatType: 'answer',
autoActivateHeartbeat: false,
convertImagesToLinks: false,
noModals: true,
showLowRepImageUploadWarning: true,
reputationToPostImages: null,
bindNavPrevention: true,
postfix: "",
imageUploader: {
brandingHtml: "Powered by u003ca class="icon-imgur-white" href="https://imgur.com/"u003eu003c/au003e",
contentPolicyHtml: "User contributions licensed under u003ca href="https://creativecommons.org/licenses/by-sa/3.0/"u003ecc by-sa 3.0 with attribution requiredu003c/au003e u003ca href="https://stackoverflow.com/legal/content-policy"u003e(content policy)u003c/au003e",
allowUrls: true
},
noCode: true, onDemand: true,
discardSelector: ".discard-answer"
,immediatelyShowMarkdownHelp:true
});
}
});
Soda is a new contributor. Be nice, and check out our Code of Conduct.
Sign up or log in
StackExchange.ready(function () {
StackExchange.helpers.onClickDraftSave('#login-link');
});
Sign up using Google
Sign up using Facebook
Sign up using Email and Password
Post as a guest
Required, but never shown
StackExchange.ready(
function () {
StackExchange.openid.initPostLogin('.new-post-login', 'https%3a%2f%2fphysics.stackexchange.com%2fquestions%2f465642%2fhow-does-ehrenfests-theorem-apply-to-the-quantum-harmonic-oscillator%23new-answer', 'question_page');
}
);
Post as a guest
Required, but never shown
3 Answers
3
active
oldest
votes
3 Answers
3
active
oldest
votes
active
oldest
votes
active
oldest
votes
$begingroup$
It is actually true, in an almost trivial way. The Ehrenfest theorem states that,
begin{equation}
frac{d}{dt}langle xrangle=langle prangle,quad frac{d}{dt}langle prangle =- langle V'(x)rangle
end{equation}
However for all eigenfunctions for the Harmonic oscillator $langle xrangle=0$ (and therefore $langle V'(x)rangle=0$) and $langle prangle=0$. So the Ehrenfest theorem on the eigenstates reduces to $0=0$.
You can see that the general version of the Ehrenfest theorem works trivially for all eigenstates. It states that for the arbitrary observable $A$ its expectation value satisfies the equation,
begin{equation}
frac{d}{dt} langle Arangle=frac{1}{ihbar}langle [A,H]rangle+langle frac{partial A}{partial t}rangle
end{equation}
However on the eigenstates,
begin{equation}
langlepsi_n| [A,H]|psi_nrangle=langlepsi_n|AH-HA|psi_nrangle=E_nlanglepsi_n|A-A|psi_nrangle=0
end{equation}
So the expectation value of the observable that doesn't explicitly depend on time, doesn't evolve on the eigenstates which is what you would expect.
So where does the Ehrenfest theorem lead to the classical dynamics? You need to consider the localized wavepackets. The simplest example would be the coherent state of the Harmonic oscillator that is the Gaussian wavepacket that follows the classical trajectory
For the Harmonic oscillator the Ehrenfest theorem is always "classical" if only in a trivial way (as in case of the eigenstates). However in general the Ehrenfest theorem reduces to the classical equation of motion only on such localized wavepackets that concentrate near the classical trajectory as $hbar$ goes to zero. The key point happens to be the interchange $langle V'(x)rangle mapsto V'(langle xrangle)$ that on general states can't be done. So if you want to recover some classical dynamics from the quantum theory look on the localized wavepackets.
$endgroup$
add a comment |
$begingroup$
It is actually true, in an almost trivial way. The Ehrenfest theorem states that,
begin{equation}
frac{d}{dt}langle xrangle=langle prangle,quad frac{d}{dt}langle prangle =- langle V'(x)rangle
end{equation}
However for all eigenfunctions for the Harmonic oscillator $langle xrangle=0$ (and therefore $langle V'(x)rangle=0$) and $langle prangle=0$. So the Ehrenfest theorem on the eigenstates reduces to $0=0$.
You can see that the general version of the Ehrenfest theorem works trivially for all eigenstates. It states that for the arbitrary observable $A$ its expectation value satisfies the equation,
begin{equation}
frac{d}{dt} langle Arangle=frac{1}{ihbar}langle [A,H]rangle+langle frac{partial A}{partial t}rangle
end{equation}
However on the eigenstates,
begin{equation}
langlepsi_n| [A,H]|psi_nrangle=langlepsi_n|AH-HA|psi_nrangle=E_nlanglepsi_n|A-A|psi_nrangle=0
end{equation}
So the expectation value of the observable that doesn't explicitly depend on time, doesn't evolve on the eigenstates which is what you would expect.
So where does the Ehrenfest theorem lead to the classical dynamics? You need to consider the localized wavepackets. The simplest example would be the coherent state of the Harmonic oscillator that is the Gaussian wavepacket that follows the classical trajectory
For the Harmonic oscillator the Ehrenfest theorem is always "classical" if only in a trivial way (as in case of the eigenstates). However in general the Ehrenfest theorem reduces to the classical equation of motion only on such localized wavepackets that concentrate near the classical trajectory as $hbar$ goes to zero. The key point happens to be the interchange $langle V'(x)rangle mapsto V'(langle xrangle)$ that on general states can't be done. So if you want to recover some classical dynamics from the quantum theory look on the localized wavepackets.
$endgroup$
add a comment |
$begingroup$
It is actually true, in an almost trivial way. The Ehrenfest theorem states that,
begin{equation}
frac{d}{dt}langle xrangle=langle prangle,quad frac{d}{dt}langle prangle =- langle V'(x)rangle
end{equation}
However for all eigenfunctions for the Harmonic oscillator $langle xrangle=0$ (and therefore $langle V'(x)rangle=0$) and $langle prangle=0$. So the Ehrenfest theorem on the eigenstates reduces to $0=0$.
You can see that the general version of the Ehrenfest theorem works trivially for all eigenstates. It states that for the arbitrary observable $A$ its expectation value satisfies the equation,
begin{equation}
frac{d}{dt} langle Arangle=frac{1}{ihbar}langle [A,H]rangle+langle frac{partial A}{partial t}rangle
end{equation}
However on the eigenstates,
begin{equation}
langlepsi_n| [A,H]|psi_nrangle=langlepsi_n|AH-HA|psi_nrangle=E_nlanglepsi_n|A-A|psi_nrangle=0
end{equation}
So the expectation value of the observable that doesn't explicitly depend on time, doesn't evolve on the eigenstates which is what you would expect.
So where does the Ehrenfest theorem lead to the classical dynamics? You need to consider the localized wavepackets. The simplest example would be the coherent state of the Harmonic oscillator that is the Gaussian wavepacket that follows the classical trajectory
For the Harmonic oscillator the Ehrenfest theorem is always "classical" if only in a trivial way (as in case of the eigenstates). However in general the Ehrenfest theorem reduces to the classical equation of motion only on such localized wavepackets that concentrate near the classical trajectory as $hbar$ goes to zero. The key point happens to be the interchange $langle V'(x)rangle mapsto V'(langle xrangle)$ that on general states can't be done. So if you want to recover some classical dynamics from the quantum theory look on the localized wavepackets.
$endgroup$
It is actually true, in an almost trivial way. The Ehrenfest theorem states that,
begin{equation}
frac{d}{dt}langle xrangle=langle prangle,quad frac{d}{dt}langle prangle =- langle V'(x)rangle
end{equation}
However for all eigenfunctions for the Harmonic oscillator $langle xrangle=0$ (and therefore $langle V'(x)rangle=0$) and $langle prangle=0$. So the Ehrenfest theorem on the eigenstates reduces to $0=0$.
You can see that the general version of the Ehrenfest theorem works trivially for all eigenstates. It states that for the arbitrary observable $A$ its expectation value satisfies the equation,
begin{equation}
frac{d}{dt} langle Arangle=frac{1}{ihbar}langle [A,H]rangle+langle frac{partial A}{partial t}rangle
end{equation}
However on the eigenstates,
begin{equation}
langlepsi_n| [A,H]|psi_nrangle=langlepsi_n|AH-HA|psi_nrangle=E_nlanglepsi_n|A-A|psi_nrangle=0
end{equation}
So the expectation value of the observable that doesn't explicitly depend on time, doesn't evolve on the eigenstates which is what you would expect.
So where does the Ehrenfest theorem lead to the classical dynamics? You need to consider the localized wavepackets. The simplest example would be the coherent state of the Harmonic oscillator that is the Gaussian wavepacket that follows the classical trajectory
For the Harmonic oscillator the Ehrenfest theorem is always "classical" if only in a trivial way (as in case of the eigenstates). However in general the Ehrenfest theorem reduces to the classical equation of motion only on such localized wavepackets that concentrate near the classical trajectory as $hbar$ goes to zero. The key point happens to be the interchange $langle V'(x)rangle mapsto V'(langle xrangle)$ that on general states can't be done. So if you want to recover some classical dynamics from the quantum theory look on the localized wavepackets.
answered 15 hours ago
OONOON
4,5792816
4,5792816
add a comment |
add a comment |
$begingroup$
- Your version of $psi$ is (I'm sure you know) derived from the Time Independent Schrodinger equation, $hat{H}psi=Epsi$. To find time-dependent solutions, we solve $$ifrac{partial}{partial t}psi=hat{H}psi.$$ You were trying to solve for stationary states, and the entire point of those is that $|psi(x)|^2$ does not change over time. Still, for these stationary solutions solutions,$$frac{mathrm{d}}{mathrm{d}t}left<xright>=left<pright>=0; frac{mathrm{d}}{mathrm{d}t}left<pright>=-left<frac{mathrm{d}}{mathrm{d}x}V (x)right>=0,$$ which is in accordance with the Ehrenfest theorem (albeit uninformatively).
Be careful about the time dependence of your reported $psi$: you're actually dealing with $Psi(x, t)=psi(x)e^{-itE/hbar}$ for those stationary states. Of course, this doesn't relate to the Ehrenfest theorem, but it's something worth mentioning: the complex and real parts are oscillating, as shown by the pink and blue lines in this diagram (from the wikipedia page on QHO):
Do not make the mistake of suggesting that all derivatives with respect to time are necessarily automatically equal to zero because the wavefunction is time-independent. Contrary to what the shorthand notation suggests, we do have a (separable) time-dependent bit.
Referring to the same diagram, observe parts G and H: these represent coherent states, which can be understood using Ehrenfest's theorem because $|psi|^2$ looks like a Gaussian which follows a classical $sin$ or $cos$ function.
See
Kanasugi, H., and H. Okada. “Systematic Treatment of General Time-Dependent Harmonic Oscillator in Classical and Quantum Mechanics.” Progress of Theoretical Physics, vol. 93, no. 5, 1995, pp. 949–960., doi:10.1143/ptp/93.5.949.
$endgroup$
add a comment |
$begingroup$
- Your version of $psi$ is (I'm sure you know) derived from the Time Independent Schrodinger equation, $hat{H}psi=Epsi$. To find time-dependent solutions, we solve $$ifrac{partial}{partial t}psi=hat{H}psi.$$ You were trying to solve for stationary states, and the entire point of those is that $|psi(x)|^2$ does not change over time. Still, for these stationary solutions solutions,$$frac{mathrm{d}}{mathrm{d}t}left<xright>=left<pright>=0; frac{mathrm{d}}{mathrm{d}t}left<pright>=-left<frac{mathrm{d}}{mathrm{d}x}V (x)right>=0,$$ which is in accordance with the Ehrenfest theorem (albeit uninformatively).
Be careful about the time dependence of your reported $psi$: you're actually dealing with $Psi(x, t)=psi(x)e^{-itE/hbar}$ for those stationary states. Of course, this doesn't relate to the Ehrenfest theorem, but it's something worth mentioning: the complex and real parts are oscillating, as shown by the pink and blue lines in this diagram (from the wikipedia page on QHO):
Do not make the mistake of suggesting that all derivatives with respect to time are necessarily automatically equal to zero because the wavefunction is time-independent. Contrary to what the shorthand notation suggests, we do have a (separable) time-dependent bit.
Referring to the same diagram, observe parts G and H: these represent coherent states, which can be understood using Ehrenfest's theorem because $|psi|^2$ looks like a Gaussian which follows a classical $sin$ or $cos$ function.
See
Kanasugi, H., and H. Okada. “Systematic Treatment of General Time-Dependent Harmonic Oscillator in Classical and Quantum Mechanics.” Progress of Theoretical Physics, vol. 93, no. 5, 1995, pp. 949–960., doi:10.1143/ptp/93.5.949.
$endgroup$
add a comment |
$begingroup$
- Your version of $psi$ is (I'm sure you know) derived from the Time Independent Schrodinger equation, $hat{H}psi=Epsi$. To find time-dependent solutions, we solve $$ifrac{partial}{partial t}psi=hat{H}psi.$$ You were trying to solve for stationary states, and the entire point of those is that $|psi(x)|^2$ does not change over time. Still, for these stationary solutions solutions,$$frac{mathrm{d}}{mathrm{d}t}left<xright>=left<pright>=0; frac{mathrm{d}}{mathrm{d}t}left<pright>=-left<frac{mathrm{d}}{mathrm{d}x}V (x)right>=0,$$ which is in accordance with the Ehrenfest theorem (albeit uninformatively).
Be careful about the time dependence of your reported $psi$: you're actually dealing with $Psi(x, t)=psi(x)e^{-itE/hbar}$ for those stationary states. Of course, this doesn't relate to the Ehrenfest theorem, but it's something worth mentioning: the complex and real parts are oscillating, as shown by the pink and blue lines in this diagram (from the wikipedia page on QHO):
Do not make the mistake of suggesting that all derivatives with respect to time are necessarily automatically equal to zero because the wavefunction is time-independent. Contrary to what the shorthand notation suggests, we do have a (separable) time-dependent bit.
Referring to the same diagram, observe parts G and H: these represent coherent states, which can be understood using Ehrenfest's theorem because $|psi|^2$ looks like a Gaussian which follows a classical $sin$ or $cos$ function.
See
Kanasugi, H., and H. Okada. “Systematic Treatment of General Time-Dependent Harmonic Oscillator in Classical and Quantum Mechanics.” Progress of Theoretical Physics, vol. 93, no. 5, 1995, pp. 949–960., doi:10.1143/ptp/93.5.949.
$endgroup$
- Your version of $psi$ is (I'm sure you know) derived from the Time Independent Schrodinger equation, $hat{H}psi=Epsi$. To find time-dependent solutions, we solve $$ifrac{partial}{partial t}psi=hat{H}psi.$$ You were trying to solve for stationary states, and the entire point of those is that $|psi(x)|^2$ does not change over time. Still, for these stationary solutions solutions,$$frac{mathrm{d}}{mathrm{d}t}left<xright>=left<pright>=0; frac{mathrm{d}}{mathrm{d}t}left<pright>=-left<frac{mathrm{d}}{mathrm{d}x}V (x)right>=0,$$ which is in accordance with the Ehrenfest theorem (albeit uninformatively).
Be careful about the time dependence of your reported $psi$: you're actually dealing with $Psi(x, t)=psi(x)e^{-itE/hbar}$ for those stationary states. Of course, this doesn't relate to the Ehrenfest theorem, but it's something worth mentioning: the complex and real parts are oscillating, as shown by the pink and blue lines in this diagram (from the wikipedia page on QHO):
Do not make the mistake of suggesting that all derivatives with respect to time are necessarily automatically equal to zero because the wavefunction is time-independent. Contrary to what the shorthand notation suggests, we do have a (separable) time-dependent bit.
Referring to the same diagram, observe parts G and H: these represent coherent states, which can be understood using Ehrenfest's theorem because $|psi|^2$ looks like a Gaussian which follows a classical $sin$ or $cos$ function.
See
Kanasugi, H., and H. Okada. “Systematic Treatment of General Time-Dependent Harmonic Oscillator in Classical and Quantum Mechanics.” Progress of Theoretical Physics, vol. 93, no. 5, 1995, pp. 949–960., doi:10.1143/ptp/93.5.949.
edited 11 hours ago
answered 15 hours ago
ChairChair
4,33772240
4,33772240
add a comment |
add a comment |
$begingroup$
What you wrote down is just a complete set of solutions of the time independent Schrödinger equation.
begin{align}
[- frac{hbar²}{2m} Delta + V(x) ] Psi(x) = E Psi(x)
end{align}
Of course this solutions don't carry any time dependence, because the time-independent Schrödinger equation (in this representation) only makes statements about functions that depend on the spatial coordinates only (in this case, this is only x).
How to get to the time-dependent solutions?
A particular property of the solutions $Psi_{E}$ of the time-independent Schrödinger Equation is that $Psi_{E}(x) e^{-i frac{E}{hbar}t}$ is a solution to the time-dependent Schrödinger equation.
If you would apply this to your suggested set of solutions of the harmonic oscillator, you would arrive at time dependent solutions. THOSE are the ones that the Ehrenfest-Theorem makes a statement about.
You would then calculate that the expectation values $<X>$ and $<P>$ do not change. But you would as well calculate that both of this values are 0. This is in perfect agreement with the Theorems of Ehrenfest, and the classical analogon would be a particle resting at the deepest spot of the harmonic potential.
$endgroup$
add a comment |
$begingroup$
What you wrote down is just a complete set of solutions of the time independent Schrödinger equation.
begin{align}
[- frac{hbar²}{2m} Delta + V(x) ] Psi(x) = E Psi(x)
end{align}
Of course this solutions don't carry any time dependence, because the time-independent Schrödinger equation (in this representation) only makes statements about functions that depend on the spatial coordinates only (in this case, this is only x).
How to get to the time-dependent solutions?
A particular property of the solutions $Psi_{E}$ of the time-independent Schrödinger Equation is that $Psi_{E}(x) e^{-i frac{E}{hbar}t}$ is a solution to the time-dependent Schrödinger equation.
If you would apply this to your suggested set of solutions of the harmonic oscillator, you would arrive at time dependent solutions. THOSE are the ones that the Ehrenfest-Theorem makes a statement about.
You would then calculate that the expectation values $<X>$ and $<P>$ do not change. But you would as well calculate that both of this values are 0. This is in perfect agreement with the Theorems of Ehrenfest, and the classical analogon would be a particle resting at the deepest spot of the harmonic potential.
$endgroup$
add a comment |
$begingroup$
What you wrote down is just a complete set of solutions of the time independent Schrödinger equation.
begin{align}
[- frac{hbar²}{2m} Delta + V(x) ] Psi(x) = E Psi(x)
end{align}
Of course this solutions don't carry any time dependence, because the time-independent Schrödinger equation (in this representation) only makes statements about functions that depend on the spatial coordinates only (in this case, this is only x).
How to get to the time-dependent solutions?
A particular property of the solutions $Psi_{E}$ of the time-independent Schrödinger Equation is that $Psi_{E}(x) e^{-i frac{E}{hbar}t}$ is a solution to the time-dependent Schrödinger equation.
If you would apply this to your suggested set of solutions of the harmonic oscillator, you would arrive at time dependent solutions. THOSE are the ones that the Ehrenfest-Theorem makes a statement about.
You would then calculate that the expectation values $<X>$ and $<P>$ do not change. But you would as well calculate that both of this values are 0. This is in perfect agreement with the Theorems of Ehrenfest, and the classical analogon would be a particle resting at the deepest spot of the harmonic potential.
$endgroup$
What you wrote down is just a complete set of solutions of the time independent Schrödinger equation.
begin{align}
[- frac{hbar²}{2m} Delta + V(x) ] Psi(x) = E Psi(x)
end{align}
Of course this solutions don't carry any time dependence, because the time-independent Schrödinger equation (in this representation) only makes statements about functions that depend on the spatial coordinates only (in this case, this is only x).
How to get to the time-dependent solutions?
A particular property of the solutions $Psi_{E}$ of the time-independent Schrödinger Equation is that $Psi_{E}(x) e^{-i frac{E}{hbar}t}$ is a solution to the time-dependent Schrödinger equation.
If you would apply this to your suggested set of solutions of the harmonic oscillator, you would arrive at time dependent solutions. THOSE are the ones that the Ehrenfest-Theorem makes a statement about.
You would then calculate that the expectation values $<X>$ and $<P>$ do not change. But you would as well calculate that both of this values are 0. This is in perfect agreement with the Theorems of Ehrenfest, and the classical analogon would be a particle resting at the deepest spot of the harmonic potential.
answered 15 hours ago
QuantumwhispQuantumwhisp
2,748724
2,748724
add a comment |
add a comment |
Soda is a new contributor. Be nice, and check out our Code of Conduct.
Soda is a new contributor. Be nice, and check out our Code of Conduct.
Soda is a new contributor. Be nice, and check out our Code of Conduct.
Soda is a new contributor. Be nice, and check out our Code of Conduct.
Thanks for contributing an answer to Physics Stack Exchange!
- Please be sure to answer the question. Provide details and share your research!
But avoid …
- Asking for help, clarification, or responding to other answers.
- Making statements based on opinion; back them up with references or personal experience.
Use MathJax to format equations. MathJax reference.
To learn more, see our tips on writing great answers.
Sign up or log in
StackExchange.ready(function () {
StackExchange.helpers.onClickDraftSave('#login-link');
});
Sign up using Google
Sign up using Facebook
Sign up using Email and Password
Post as a guest
Required, but never shown
StackExchange.ready(
function () {
StackExchange.openid.initPostLogin('.new-post-login', 'https%3a%2f%2fphysics.stackexchange.com%2fquestions%2f465642%2fhow-does-ehrenfests-theorem-apply-to-the-quantum-harmonic-oscillator%23new-answer', 'question_page');
}
);
Post as a guest
Required, but never shown
Sign up or log in
StackExchange.ready(function () {
StackExchange.helpers.onClickDraftSave('#login-link');
});
Sign up using Google
Sign up using Facebook
Sign up using Email and Password
Post as a guest
Required, but never shown
Sign up or log in
StackExchange.ready(function () {
StackExchange.helpers.onClickDraftSave('#login-link');
});
Sign up using Google
Sign up using Facebook
Sign up using Email and Password
Post as a guest
Required, but never shown
Sign up or log in
StackExchange.ready(function () {
StackExchange.helpers.onClickDraftSave('#login-link');
});
Sign up using Google
Sign up using Facebook
Sign up using Email and Password
Sign up using Google
Sign up using Facebook
Sign up using Email and Password
Post as a guest
Required, but never shown
Required, but never shown
Required, but never shown
Required, but never shown
Required, but never shown
Required, but never shown
Required, but never shown
Required, but never shown
Required, but never shown
$begingroup$
Maybe this question of mine can answer your question: physics.stackexchange.com/questions/267835/…
$endgroup$
– Quantumwhisp
15 hours ago
$begingroup$
Though it does not address the question asked, I used to have students plot the position probability distribution for the quantum and classical cases over a range of energies corresponding to, say, $n = 1,2,3,4,10$ and a "big" number like $20$. I'll post a figure in chat: chat.stackexchange.com/transcript/message/49407141#49407141.
$endgroup$
– dmckee♦
2 hours ago